Rational expression limit problem

Click For Summary
The discussion focuses on proving the limits of rational expressions involving polynomials as x approaches infinity. It is established that when two polynomials of the same degree have the same leading coefficient, their limit approaches one. Conversely, if the degree of the denominator is greater than that of the numerator, the limit approaches zero. Examples are provided to illustrate that limits can vary based on the leading coefficients and degrees of the polynomials involved. The user seeks assistance in understanding induction as a method for proving these limits.
StonedPanda
Messages
60
Reaction score
0
Sligtly more complex than the average one, I'd assume. How would I go about proving that the limit of of a rational expression consisting of two polynomials of the same degree goes to one and the limit of one where the degree of the bottom is greater than the degree of the top goes to zero. I'd imagine i'd have to use induction, but I've never learned it!

This step is actually a step in a more complicated problem, which I've got for the most part.
 
Physics news on Phys.org
Limit as x approaches infinity? Consider:

2x/x

this is a rational function of two polynomials of the same degree whose limit is never 1 for any x, it is always 2. Also consider:

2(x+1)/x

again, it has a limit of 2 as x approaches infinity, but it's limit isn't always 2, for example, it is 4 when x approaches 1. Perhaps you mean when both polynomials also have the same leading coefficient. In that case, the limit as x approaches infinity will be 1.
 
yes, sorry, i meant for the case when the polynomials has the same leading coefficient as the variable approaches infinity.
 
so can anyone help me prove this? I'm trying to teach myself induction...
 
Question: A clock's minute hand has length 4 and its hour hand has length 3. What is the distance between the tips at the moment when it is increasing most rapidly?(Putnam Exam Question) Answer: Making assumption that both the hands moves at constant angular velocities, the answer is ## \sqrt{7} .## But don't you think this assumption is somewhat doubtful and wrong?

Similar threads

  • · Replies 14 ·
Replies
14
Views
2K
  • · Replies 11 ·
Replies
11
Views
2K
  • · Replies 3 ·
Replies
3
Views
1K
Replies
10
Views
2K
  • · Replies 3 ·
Replies
3
Views
3K
Replies
13
Views
6K
  • · Replies 1 ·
Replies
1
Views
2K
  • · Replies 21 ·
Replies
21
Views
2K
  • · Replies 2 ·
Replies
2
Views
2K
Replies
32
Views
3K